The Student Room Group

Please check if my solution for differential eq is correct

And if its not correct please point out where i went wrong.
Reply 1
Original post by Jyashi
And if its not correct please point out where i went wrong.

15pp2=15p1p2\displaystyle \frac{1}{5p-p^2} = \frac{1}{5p} - \frac{1}{p^2}

This is incorrect - you can't split a fraction up like this. In general :

AB+CAB+AC\displaystyle \frac{A}{B+C} \neq \frac{A}{B} + \frac{A}{C}


You should be using partial fractions instead.
Nope you cannot split up the integral on the LHS by splitting its denominator like that
Reply 3
I did have a doubt about that when i did it. So partial fractions are one option. Are there any other options of doing this without multiplying P with 5-P?

Original post by notnek
15pp2=15p1p2\displaystyle \frac{1}{5p-p^2} = \frac{1}{5p} - \frac{1}{p^2}

This is incorrect - you can't split a fraction up like this. In general :

AB+CAB+AC\displaystyle \frac{A}{B+C} \neq \frac{A}{B} + \frac{A}{C}


You should be using partial fractions instead.
Reply 4
Original post by Jyashi
I did have a doubt about that when i did it. So partial fractions are one option. Are there any other options of doing this without multiplying P with 5-P?

There are other methods but they go beyond C4 and would make this more complicated than it is.

You should be using partial fractions for a differential equation like this.
Reply 5
Ok thanks ill give it a second go and post back solution on this thread.

Original post by notnek
There are other methods but they go beyond C4 and would make this more complicated than it is.

You should be using partial fractions for a differential equation like this.
Reply 6
After going through the whole process again i get:

LN (P (5-P))=T÷3

Is this right?

Original post by Jyashi
Ok thanks ill give it a second go and post back solution on this thread.
(edited 7 years ago)
Reply 7
After going through the whole process again i get:

LN (P (5-P))=T÷3

Is this right?
Original post by notnek
There are other methods but they go beyond C4 and would make this more complicated than it is.

You should be using partial fractions for a differential equation like this.
Reply 8
Original post by Jyashi
After going through the whole process again i get:

LN (P (5-P))=T÷3

Is this right?

It's close but not quite right.

Can you post your working?
Reply 9
I realised another mistake and have the right answer now which is LN (P)÷5 - LN (P-5) ÷ 5 = t÷15


Original post by notnek
It's close but not quite right.

Can you post your working?

Quick Reply

Latest